• Shuffle
    Toggle On
    Toggle Off
  • Alphabetize
    Toggle On
    Toggle Off
  • Front First
    Toggle On
    Toggle Off
  • Both Sides
    Toggle On
    Toggle Off
  • Read
    Toggle On
    Toggle Off
Reading...
Front

Card Range To Study

through

image

Play button

image

Play button

image

Progress

1/63

Click to flip

Use LEFT and RIGHT arrow keys to navigate between flashcards;

Use UP and DOWN arrow keys to flip the card;

H to show hint;

A reads text to speech;

63 Cards in this Set

  • Front
  • Back
BF is a 42 year old man with type 1 diabetes, hypertension, and chronic kidney disease who presents with the following:

Labs Na 135 meq/L Ca 10.2 mg/dl
K 6.2 meq/L PO4 2.6 mg/dl
ECG: peaked T waves; prolonged P-R interval; bradycardia

Which of the following is the most appropriate first pharmacotherapeutic intervention for this patient?

a. give intravenous furosemide 40 mg
b. give intravenous Ca gluconate
c. give oral sevelamer
d. give oral sodium polystyrene suffocate
b. give intravenous Ca gluconate

Please see objective b, slide #33. Patient has hyperkalemia with ECG changes and Ca gluconate should be given first to stabilize myocardial conduction even though his serum Ca is quite high. Intravenous furosemide and/or Kayexalate (selection d) can follow. Oral sevelamer would be useful for this patients hyperphosphatemia but its use should not precede administration of intravenous Ca.
BF is a 42 year old man with type 1 diabetes, hypertension, and chronic kidney disease who presents with the following:

Labs Na 135 meq/L Ca 10.2 mg/dl
K 6.2 meq/L PO4 2.6 mg/dl
ECG: peaked T waves; prolonged P-R interval; bradycardia


Which of the following most closely approximates the appropriate administration of sodium polystyrene sulfonate to this patient?

a. 15 Gm po q 4 hr x 3 doses
b. 45 Gm po x 1 dose only
c. 60 Gm po q 4 hr x 3 doses
d. 180 Gm po q 4 hr x 1 dose only
c. c. 60 Gm po q 4 hr x 3 doses

Please see objective 2, slide 34, and section entitled Hypokalemia in Chapter 54 of required reading: Enough Kayexalate must be given to capture at least 100 meq of K to lower this patient’s serum K to < 5.0 meq/L. Additionally, the oral dose of Kayexalate must be given in increments. Selections a, b, and d are incorrect as a consequence.
DH is a 73 year old man with diabetes, chronic kidney disease, osteoarthritis, and hypertension hospitalized with a urinary tract infection. His current medications include Metformin, Ibuprofen, and Lisinopril.

Which of the following most closely describes the number of risk factors this patient has for development of acute renal failure?

a. three
b. four
c. five
d. six
d. six

Please see objective 1, slides 5 and 7, and sections entitled Epidemiology and Etiology of Chapter 45 of required reading. Risk factors in this patient are age > 60, diabetes, pre-existing renal disease, infection, hospitalization, and use of nephrotoxins (i.e. metformin, ibuprofen, and possibly lisinopril).
DH is a 73 year old man with diabetes, chronic kidney disease, osteoarthritis, and hypertension hospitalized with a urinary tract infection. His current medications include Metformin, Ibuprofen, and Lisinopril.

During this hospitalization it became necessary for the patient to undergo an elective cardiac catheterization. Since he will be receiving contrast media as part of this catheterization, which of the following would be LEAST useful for prevention of contrast media induced acute renal failure?

a. discontinue all nephrotoxins prior to catheterization
b. intravenous furosemide 40 mg twice daily the day before, day of, and day after the catheterization
c. Sodium acetylcysteine 600 mg by mouth twice daily the day before and day of catheterization
d. Intravenous normal saline the day before and day of catheterization
b. intravenous furosemide 40 mg twice daily the day before, day of, and day after the catheterization

Please see objective 4, slides 20 and 21, and section entitled Pharmacologic Therapies of Prevention and Treatment of Acute Renal Failure in Chapter 45 of required reading. A diuretic is most likely to cause acute renal failure. Selections a, c, and d are reasonable strategies for prevention of acute renal failure.
DH is a 73 year old man with diabetes, chronic kidney disease, osteoarthritis, and hypertension hospitalized with a urinary tract infection. His current medications include Metformin, Ibuprofen, and Lisinopril.

In spite of your best efforts, DH develops acute renal failure following the catheterization. Which of the following is the best management strategy for this patient now?

a. intravenous furosemide especially if patient is hypovolemic
b. fluid restriction if hypovolemic
c. hemodialysis
d. combined use of metolazone and furosemide if patient is hypovolemic
c. hemodialysis

Please see objective 5, slides 24 and 25, and section entitled Management of Established Acute Renal Failure in Chapter 45 of required reading. Furosemide, alone or together with metolazone, would be reasonable if the patient were hypervolemic. Their use otherwise would likely exacerbate the acute renal failure. Similarly, fluid restriction would do the same and would be appropriate, once again, if the patient were hypervolemic.
(refer to CASE A)

Which of the following most closely characterize the clinical stage of this patient’s renal function?

a. patient has normal renal function
b. stage 5
c. stage 3
d. stage 1
b. stage 5

In this patient CLcr =(140 – 52)(72) / (72) = 14.6 ml/min.
This level of renal (dys)function is consistent with level 5.
(Refer to CASE A)

Which of the following conditions present in this patient are most likely to result in exacerbation of this patient’s renal dysfunction?

a. diabetes, hypertension
b. obesity, hyperlipidemia
c. low income, ethnic minority
d. tobacco use, hyperlipidemia
a. diabetes, hypertension

Please see objective #3, slide 11 of Chronic Renal Failure presentation. Selection c lists susceptibility factors which are useful for characterizing risk if both were true of this patient. Selections b and d are all either initiation factors or progression factors but this patient is not obese and there was no mention of tobacco use in this case description.
(Refer to CASE A)

Which of the following would be most useful for determining whether or not this patient’s anemia is microcytic?

a. serum folic acid concentration
b. serum Fe concentration
c. transferring saturation
d. serum vitamin B12 concentration
c. transferring saturation

Please see objectives 3 thru 6, slides 43 thru 47 and 50, and section entitled Anemia of Chronic Kidney Disease (in Treatment: Chronic Kidney Disease) in Chapter 47 of required reading. Since microcytic anemia is most likely related to a deficiency of Fe, only selection c provides the necessary information to determine whether or not this patient’s anemia is microcytic.
(Refer to CASE A)

Which of the following signs/symptoms would, if present in this patient, be consistent with renal osteodystrophy?

a. chest pain, shortness of breath
b. bone pain, myopathy
c. pulmonary fibrosis, pulmonary hypertension
d. all of the above
d. all of the above

Please see objective 4, slide 29 of Chronic Renal Failure presentation, and section entitled Secondary Hyperparathyroidism and Renal Osteodystrophy (in Pathophysiology) of Chapter 47 of required reading.
(Refer to CASE A)

Which of the following would be most useful for initial management of this patient’s renal osteodystrophy?

a. Calcium carbonate
b. Lanthanum carbonate
c. Magnesium/Aluminum hydroxide gel
d. Cinacalcet
b. Lanthanum carbonate

Please see objectives 5 and 6, slide 32, and section in Chapter 47 of required reading under “Treatment: Chronic Kidney Disease” entitled Secondary Hyperparathyroidism and Renal Osteodystrophy. Since this patient is already hypercalcemic and hypermagnesemic, selections a and c are incorrect. Selection d would be useful for reducing PTH concentrations later on.
(Refer to CASE A)

Which of the following is most correct about this patient’s systemic acidosis?

a. there is no evidence that this patient has acidosis
b. this is beneficial since it protects against renal osteodystrophy
c. this is detrimental since it exacerbates renal osteodystrophy
d. it is beneficial in that it increases Vit D synthesis
c. this is detrimental since it exacerbates renal osteodystrophy

Please see objectives 3 and 4, slides 26 of Chronic Renal Failure presentation, and section in Chapter 47 of required reading entitled Chronic Metabolic Acidosis in Treatment in Chronic Kidney Disease. Selection a is incorrect since the patient’s bicarbonate level is below normal and consistent with acidosis. Selection b and d are both incorrect since acidosis is associated with the opposite phenomena.
(Refer to Case B)

Ms. LPG’s physician wishes to initiate corrective therapy for the hypokalemia.
His goal of therapy is a serum K+ of 4.0 meq/L. Which of the following would
be the best management strategy for this patient at this time?
a. Give 150 meq KCl intravenously over 1 - 2 minutes
b. Give 150 meq K Phosphate intravenously over 1 - 2 minutes
c. Give 10 meq of KCl (mixed in 0.9% NaCl solution) intravenously hourly
for 15 hrs
d. Give 10 meq of KCl (mixed in 5% Dextrose solution) intravenously hourly for
15 hrs
c. Give 10 meq of KCl (mixed in 0.9% NaCl solution) intravenously hourly
for 15 hrs

Please see objective #2. Selection “a” and “b” are both incorrect due to the
rapidity of administration. Parenteral K+ (whether as chloride or phosphate) is always
administered slowly (see slides 22 and page 971 of text). Selection d is incorrect since
dextrose administration will likely drive administered K+ intracellularly and delay
correction of the problem.
(Refer to Case B)

Which of the following is the most likely cause of this patient’s hypercalcemia?
a. Use of a diuretic such as furosemide, a loop diuretic
b. Use of a diuretic such as hydrochlorothiazide (or HCTZ), a thiazide
diuretic
c. Inadequate Vit D intake
d. This patient does not have hypercalcemia.
b. Use of a diuretic such as hydrochlorothiazide (or HCTZ), a thiazide
diuretic

Please see objective #1. Use of furosemide is associated with hypocalcemia
(selection “a” incorrect; see slide #30 and pages 953 and 956 of text). Inadequate vitamin
D intake would likewise be associated with hypocalcemia (selection “c” incorrect; see
slide #29 and page 956 of text. Finally, selection “d” is incorrect even though the
reported Ca++ for this patient is within normal limits. By virtue of this patient’s
hypoalbuminemia, this patient’s functional Ca++ is above normal limits (10.5 mg/dl +
0.8 [4 – 2.5] = 11.7 mg/dl).
(Refer to Case B)

Shortly after administration of the correct dose of K+, a repeat serum K+
concentration is found to be 3.0 meq/L. Which of the following is most likely to
be associated with resolution of this continuing hypokalemia?
a. Give more K+
b. Change administration of KCl to K Phosphate
c. Change administration of K Phosphate to KCl
d. Initiate Mg replacement therapy.
d. Initiate Mg replacement therapy.Please see objective #2. Selection “a” is incorrect since the correct dose was
already administered (see first sentence in question). Selections “b” and “c” are
incorrect as the salt of K is not relevant to this problem. The problem (see slide #19 and
page 970 of text) is this patient is hypromagnesemia and, until Mg is replenished or
provided, serum K+ will not rise
(Refer to Case B)

Which of the following is most correct regarding this patient’s
hypophosphatemia?
a. a reasonable therapy for this problem would be a bisphosphonate although
response would be delayed
b. a reasonable therapy for this problem would be intravenous K Phosphate
c. a reasonable therapy for this problem would be sevelamer.
d. a reasonable cause of this problem in this patient is renal insufficiency
b. a reasonable therapy for this problem would be intravenous K Phosphate

especially since this patient is also hypokalemic, although administration
must be slow. Please see objective #2. Selection “a” is incorrect as it is a reasonable
therapy for hypercalcemia (see slide #37 and page 954of text) and has no role in the
therapy of hypophosphatemia. Selection “c” is incorrect as this product is used for
treatment of hyperphosphatemia (see slide #51 and page 961 of text). Selection “d” is
incorrect since this patient’s renal function is now within normal limits following her
transplantation. Renal insufficiency is more often associated with hyperphosphatemia.
(Refer to Case B)

Ms. LPG’s physician now elects to initiate replenishment of this patient’s
depleted Mg++. Which of the following would be LEAST beneficial to this
patient at this time?
a. Immediate administration of intravenous Ca gluconate
b. Mg Oxide tablets 600 mg po bid
c. MgS04 1.0 Gm intravenously
d. MgS04 1.0 Gm intramuscularly
a. Immediate administration of intravenous Ca gluconate

Please see objective #1. Such use of parental Ca++ would be appropriate for
hypermagnesemia (and hyperkalemia for that matter) but not for this patient. Selection
“b” is incorrect even though the dose is low (see slide #42) and selection “d” is incorrect
even though such administration would be quite uncomfortable. In both instances, there
is at least some benefit to the patient. Selection “c” represents the preferred
administration strategy for parenteral Mg.
KM is a 64 year old man (5 ft 10 in; 73 kg) admitted to the hospital for an elective
cardiac catheterization (with contrast media). His past medical history includes arthritis
(degenerative joint disease), diabetes, and heart failure and his list of medications
includes ibuprofen (Motrin), a non-steroidal anti-inflammatory agent, metformin
(Glucophage), furosemide (Lasix), a K-wasting diuretic, and insulin. His admission labs include the following: Na 142 meq/L, K 5.3 meq/L, Cl 110 meq/L, CO2 24 meq/L, BUN
20 mg/dl, Cr 2.0 mg/dl.

The day after his cardiac catheterization, he is found to be in acute renal failure.
Which of the following would have most likely prevented this occurrence of acute
renal failure?
a. discontinue ibuprofen, metformin, and furosemide
b. low – dose dopamine, furosemide, and fluid restriction the day before the
c. catheterization
d. administer normal saline and N-acetylcysteine the day after the catheterization
e. contrast media does not adversely affect renal function; no prophylactic
measures are necessary
a. discontinue ibuprofen, metformin, and furosemide

please see objective 5. Selection “b” is incorrect
since two of these (furosemide and fluid restriction) actually increase the likelihood of
ARF. Low dose dopamine has not been shown to work. Selection “c” is incorrect since
both of these legitimate strategies need to be administered at least the day before the
catheterization and not the day after as the selection describes. Selection “d” is incorrect.
See pages 789-790 of text for details.
KM is a 64 year old man (5 ft 10 in; 73 kg) admitted to the hospital for an elective
cardiac catheterization (with contrast media). His past medical history includes arthritis
(degenerative joint disease), diabetes, and heart failure and his list of medications
includes ibuprofen (Motrin), a non-steroidal anti-inflammatory agent, metformin
(Glucophage), furosemide (Lasix), a K-wasting diuretic, and insulin. His admission labs
include the following: Na 142 meq/L, K 5.3 meq/L, Cl 110 meq/L, CO2 24 meq/L, BUN
20 mg/dl, Cr 2.0 mg/dl.
The day after his cardiac catheterization, he is found to be in acute renal failure.

Which of the following is the most correct characterization of this patient’s acute
renal failure?
a. post renal obstruction acute renal failure
b. post renal azotemia acute renal failure
c. either functional or intrinsic acute renal failure
d. pre-renal azotemia acute renal failure
c. either functional or intrinsic acute renal failure

See slide 11 and text pages 782-785 (Table
42-2, page 783 as well). Selection “a” does not fit the clinical circumstances of this case. Selection “b”, to my knowledge, does not exist but also does not fit the clinical
circumstances. Selection “d” represents a laboratory abnormality and may precede
frank acute renal failure.
KM is a 64 year old man (5 ft 10 in; 73 kg) admitted to the hospital for an elective
cardiac catheterization (with contrast media). His past medical history includes arthritis
(degenerative joint disease), diabetes, and heart failure and his list of medications
includes ibuprofen (Motrin), a non-steroidal anti-inflammatory agent, metformin
(Glucophage), furosemide (Lasix), a K-wasting diuretic, and insulin. His admission labs include the following: Na 142 meq/L, K 5.3 meq/L, Cl 110 meq/L, CO2 24 meq/L, BUN
20 mg/dl, Cr 2.0 mg/dl.
The day after his cardiac catheterization, he is found to be in acute renal failure.
Which of the following represents the best strategy for managing this patient’s
acute renal failure?
a. fluid restriction if hypovolemic
b. increase doses of all current medications
c. aggressive use of furosemide especially if patient is anuric or oliguric
d. fluid restriction if anuric or oliguric
d. fluid restriction if anuric or oliguric

Failure to restrict fluid intake in the
presence of oliguria or anuria would rapidly lead to profound peripheral and central
edema (including pulmonary and CNS). Selection “b” is incorrect since three of the
medications the patient is taking (i.e. furosemide, metformin, and ibuprofen) are
nephrotoxic and should be discontinued until the patient recovers. Similarly, selection
“c” is incorrect as well. Selection “a” is incorrect since fluid restriction in such a
clinical setting would exacerbate the acute renal failure. If the patient were hypovolemic, then fluid administration would be appropriate. Please see pages 790-793 of text for
further details.
FJ is a 64 year old man with a long (many years) history of chronic kidney disease,
hypertension, diabetes, and asthma. During a routine follow-up clinic visit, the following
labs were noted: Na 142 meq/L, K 5.4 meq/L, Cl 105 meq/L, CO2 25 meq/L, BUN 20
mg/dl, Cr 2.5 mg/dl, Hgb 9.3 Gm/dl, Hct 28%, MCV 82 μm3, Alb 2.4 Gm/dl, Ca 10.5
mg/dl, phosphate 5.5 mg/dl, Fe 90 mcg/dl, TIBC 460 μg/dl, Transferrin sat’n 14%,
Ferritin 15 ng/ml.
The patient has no specific complaints but his vital signs are as follows: BP 170/100 mm
Hg, P 82, RR16, T 37°C. Patient is 6’0” and 185 lbs.

Which of the following most appropriately characterizes this patient’s stage of
renal dysfunction?
a. Stage 1
b. Stage 2
c. Stage 3
d. Patient has normal renal function for his age.
c. Stage 3

IBW = 50 + 2.3 (12) = 50 + 27.6 = 77.6 kg
% IBW = (84.1 kg – 77.6 kg)/ 77.6kg = 109%

Use TBW

CLCr = [(140 – 64) (84.1)] / [(2.5)(72)] = 6391.6/180 = 36 ml/min
FJ is a 64 year old man with a long (many years) history of chronic kidney disease,
hypertension, diabetes, and asthma. During a routine follow-up clinic visit, the following
labs were noted: Na 142 meq/L, K 5.4 meq/L, Cl 105 meq/L, CO2 25 meq/L, BUN 20
mg/dl, Cr 2.5 mg/dl, Hgb 9.3 Gm/dl, Hct 28%, MCV 82 μm3, Alb 2.4 Gm/dl, Ca 10.5
mg/dl, phosphate 5.5 mg/dl, Fe 90 mcg/dl, TIBC 460 μg/dl, Transferrin sat’n 14%,
Ferritin 15 ng/ml.
The patient has no specific complaints but his vital signs are as follows: BP 170/100 mm
Hg, P 82, RR16, T 37°C. Patient is 6’0” and 185 lbs.

Which of the following represents most appropriate therapy for this patient’s
metabolic acidosis?
a. patient should preferably be treated with either Shohl’s solution (citric
acid/Na citrate) or Bicitra (citric acid/Na citrate)
b. patient should preferably be treated with Polycitra (citric acid/Na citrate/K
citrate).
c. patient should preferably be treated with NaHCO3.
d. patient does not yet have metabolic acidosis; no therapy is necessary yet.
d. patient does not yet have metabolic acidosis; no therapy is necessary yet.

please see objectives 6 and 7. Patient may actually
be mildly alkalotic.
FJ is a 64 year old man with a long (many years) history of chronic kidney disease,
hypertension, diabetes, and asthma. During a routine follow-up clinic visit, the following
labs were noted: Na 142 meq/L, K 5.4 meq/L, Cl 105 meq/L, CO2 25 meq/L, BUN 20
mg/dl, Cr 2.5 mg/dl, Hgb 9.3 Gm/dl, Hct 28%, MCV 82 μm3, Alb 2.4 Gm/dl, Ca 10.5
mg/dl, phosphate 5.5 mg/dl, Fe 90 mcg/dl, TIBC 460 μg/dl, Transferrin sat’n 14%,
Ferritin 15 ng/ml.
The patient has no specific complaints but his vital signs are as follows: BP 170/100 mm
Hg, P 82, RR16, T 37°C. Patient is 6’0” and 185 lbs.

Which of the following would be most preferable for initial management of this
patient’s anemia?
a. since this patient probably has a macrocytic anemia, begin therapy with
Folic Acid.
b. since this patient probably has a macrocytic anemia, begin therapy with
vitamin B12.
c. begin therapy with erythropoietin or darbepoietin.
d. since this patient has no symptoms typically associated with anemia, no
therapy for anemia is appropriate at this time.
c. begin therapy with erythropoietin or darbepoietin

Selections “a” and “b”
are both incorrect since this patient does not have a macrocytic anemia by virtue of an
MCV that is within normal limits. Selection “d” is incorrect since absence of symptoms
is not the primary determinant of initiating therapy (see slide #19).
FJ is a 64 year old man with a long (many years) history of chronic kidney disease,
hypertension, diabetes, and asthma. During a routine follow-up clinic visit, the following
labs were noted: Na 142 meq/L, K 5.4 meq/L, Cl 105 meq/L, CO2 25 meq/L, BUN 20
mg/dl, Cr 2.5 mg/dl, Hgb 9.3 Gm/dl, Hct 28%, MCV 82 μm3, Alb 2.4 Gm/dl, Ca 10.5
mg/dl, phosphate 5.5 mg/dl, Fe 90 mcg/dl, TIBC 460 μg/dl, Transferrin sat’n 14%,
Ferritin 15 ng/ml.
The patient has no specific complaints but his vital signs are as follows: BP 170/100 mm
Hg, P 82, RR16, T 37°C. Patient is 6’0” and 185 lbs.

Which of the following describes the most appropriate use of Fe to mange this
patient’s anemia?
a. begin therapy with oral FeS04 given 2 to 3 times daily
b. since this patient’s serum Fe concentration is within normal limits, Fe therapy
should be withheld at this time.
c. since Transferrin saturation and serum Ferritin are both within normal limits,
Fe therapy should be withheld at this time.
d. do not administer Fe until resistance to other therapies has been eliminated.
a. begin therapy with oral FeS04 given 2 to 3 times daily

Selection “b” is
incorrect since serum Fe concentration is an unreliable measure of total body iron stores.
Selection “c” is incorrect even though these values are within normal limits. Please note
that both Transferrin sat’n and Ferritin are low normal. It is quite likely that initiation of
EPO or darbepoeitin (see previous question) will use up Fe stores and response to these
agents would be limited as a result. Selection “d” is incorrect since Fe stores should
ideally be optimized before initiation of EPO or darbepoeitin (see slide #19).
FJ is a 64 year old man with a long (many years) history of chronic kidney disease,
hypertension, diabetes, and asthma. During a routine follow-up clinic visit, the following
labs were noted: Na 142 meq/L, K 5.4 meq/L, Cl 105 meq/L, CO2 25 meq/L, BUN 20
mg/dl, Cr 2.5 mg/dl, Hgb 9.3 Gm/dl, Hct 28%, MCV 82 μm3, Alb 2.4 Gm/dl, Ca 10.5
mg/dl, phosphate 5.5 mg/dl, Fe 90 mcg/dl, TIBC 460 μg/dl, Transferrin sat’n 14%,
Ferritin 15 ng/ml.
The patient has no specific complaints but his vital signs are as follows: BP 170/100 mm
Hg, P 82, RR16, T 37°C. Patient is 6’0” and 185 lbs.

Which of the following most correctly describes the goal of therapy for
management of this patient’s anemia?
a. minimal to no symptoms of anemia; MCV > 100 mm3
b. minimal to no symptoms of anemia; Hgb 11 Gm/dl; Hct 33%
c. minimal to no symptoms of anemia; serum Fe 80 – 180 mcg/dl
d. minimal to no symptoms of anemia; Hgb 15 Gm/dl; Hct 42%
b. minimal to no symptoms of anemia; Hgb 11 Gm/dl; Hct 33%

please see objective 7and slide 17. Serum Fe
concentration is still an unreliable measure of anemia. Ferritin and transferring
saturation are not the monitoring parameters upon which this decision is made.
FJ is a 64 year old man with a long (many years) history of chronic kidney disease,
hypertension, diabetes, and asthma. During a routine follow-up clinic visit, the following
labs were noted: Na 142 meq/L, K 5.4 meq/L, Cl 105 meq/L, CO2 25 meq/L, BUN 20
mg/dl, Cr 2.5 mg/dl, Hgb 9.3 Gm/dl, Hct 28%, MCV 82 μm3, Alb 2.4 Gm/dl, Ca 10.5
mg/dl, phosphate 5.5 mg/dl, Fe 90 mcg/dl, TIBC 460 μg/dl, Transferrin sat’n 14%,
Ferritin 15 ng/ml.
The patient has no specific complaints but his vital signs are as follows: BP 170/100 mm
Hg, P 82, RR16, T 37°C. Patient is 6’0” and 185 lbs.
Which of the following would be most appropriate initial therapy for treatment of
this patient’s renal osteodystrophy?
a. since this patient’s serum Ca is within normal limits, he probably does not
have renal osteodystrophy yet.
b. Al/Mg hydroxide suspension (Maalox, Mylanta, others), an antacid
c. Ca CO3 (Tums, others)
d. Sevelamer (Renagel)
d. Sevelamer (Renagel)

Although the patient’s
observed Ca is within normal limits, the corrected Ca (i.e. 11.8 mg/dl) is not. Also, the
Ca – phosphate product is > 55 (i.e. 65) which strongly suggests that this patient does
have renal osteodystrophy which renders selection “a” incorrect. Selection “c” is
incorrect by virtue of this patient’s corrected hypercalcemia. Selection “b” is incorrect
since use of Mg salts is patients with chronic kidney disease may lead to
hypermagnesemia and/or aluminum toxicity.
FJ is a 64 year old man with a long (many years) history of chronic kidney disease,
hypertension, diabetes, and asthma. During a routine follow-up clinic visit, the following
labs were noted: Na 142 meq/L, K 5.4 meq/L, Cl 105 meq/L, CO2 25 meq/L, BUN 20
mg/dl, Cr 2.5 mg/dl, Hgb 9.3 Gm/dl, Hct 28%, MCV 82 μm3, Alb 2.4 Gm/dl, Ca 10.5
mg/dl, phosphate 5.5 mg/dl, Fe 90 mcg/dl, TIBC 460 μg/dl, Transferrin sat’n 14%,
Ferritin 15 ng/ml.
The patient has no specific complaints but his vital signs are as follows: BP 170/100 mm
Hg, P 82, RR16, T 37°C. Patient is 6’0” and 185 lbs.

Which of the following represent the best initial therapy for management of this
patient’s hypertension?
a. a calcium channel blocker such as verapamil (Calan) or diltiazem
(Cardizem).
a. a beta blocker such as metoprolol (Lopressor).
c. an ACE inhibitor such as ramipril (Altace) or lisinopril (Zestril)
d. hypertension is a benign condition in this patient and does not need to be
treated.
c. an ACE inhibitor such as ramipril (Altace) or lisinopril (Zestril)

please see objective 6. Selection “a” represents a reasonable
antihypertensive strategy but not for initial therapy. Selection “b” is incorrect since this
patient has asthma. Please see slides 23 and 24. I would hope that an explanation of
selection “d” would not be necessary.
FJ is a 64 year old man with a long (many years) history of chronic kidney disease,
hypertension, diabetes, and asthma. During a routine follow-up clinic visit, the following
labs were noted: Na 142 meq/L, K 5.4 meq/L, Cl 105 meq/L, CO2 25 meq/L, BUN 20
mg/dl, Cr 2.5 mg/dl, Hgb 9.3 Gm/dl, Hct 28%, MCV 82 μm3, Alb 2.4 Gm/dl, Ca 10.5
mg/dl, phosphate 5.5 mg/dl, Fe 90 mcg/dl, TIBC 460 μg/dl, Transferrin sat’n 14%,
Ferritin 15 ng/ml.
The patient has no specific complaints but his vital signs are as follows: BP 170/100 mm
Hg, P 82, RR16, T 37°C. Patient is 6’0” and 185 lbs.

Which of the following best describes appropriate use of anti-hypertensive
medication(s) for this patient?
a. a K – sparing diuretic, such as spironolactone (Aldactone), is clearly more
beneficial than a K- wasting diuretic such as furosemide (Lasix).
b. use of a beta blocker, such as metoprolol (Lopressor), is a reasonable
choice even though it may exacerbate renal dysfunction.
c. use of a beta blocker, such as metoprolol (Lopressor), is clearly
contraindicated since it may exacerbate renal dysfunction.
d. an angiotensin receptor blocker, such as candesartan (Atacand), is clearly
more beneficial than an ACE inhibitor such as ramipril (Altace).
b. use of a beta blocker, such as metoprolol (Lopressor), is a reasonable
choice even though it may exacerbate renal dysfunction.

Use of an ARB is appropriate in the
patient who is intolerant to an ACE inhibitor but it is not yet preferred over an ACE
inhibitor. Consequently, selection “d” is incorrect. Selection “a” is incorrect since a K
– sparing diuretic in this patient may exacerbate his hyperkalemia. Selection “c” is
incorrect since the benefits of beta-blockade for exceed this largely theoretical risk.
Which one of the following statements regarding the effect(s) of chronic kidney
disease on protein binding and drug distribution is/are TRUE?
a. The binding of acidic drugs (e.g., phenytoin) is typically increased and the
volume of distribution is decreased in patients with CKD.
b. The binding of acidic drugs (e.g., phenytoin) is typically decreased and the
volume of distribution is increased in patients with CKD.
c. The binding of basic drugs (e.g., quinidine) is typically increased and the
volume of distribution is decreased in patients with CKD.
d. The binding of basic drugs (e.g., quinidine) is typically decreased and the
volume of distribution is increased in patients with CKD.
e. A and C
f. A and D
g. B and C
h. B and D
g. B and C

The binding of acidic drugs is typically
decreased in patients with chronic kidney disease while binding for basic drugs is
typically increased. This has the opposite effect on the volume of distribution with the
VD being larger for acidic drugs and smaller for basic drugs
There are now a number of examples of drugs predominately metabolized that
require dosage adjustment in patients with chronic kidney disease (CKD) because
of reduced non-renal clearance. Which of the following drugs metabolized by a
CYP enzyme would you MOST expect to have decreased clearance (due to
decreased enzyme activity) in patients with CKD?
a. Chlorzoxazone (predominately metabolized by CYP2E1)
b. Omeprazole (predominately metabolized by CYP2C19)
c. Ranolazine (predominately metabolized by CYP3A4)
d. The clearance of all of these drugs would be decreased in CKD.
e. CKD will not affect the clearance of any of these drugs.
c. Ranolazine (predominately metabolized by CYP3A4)

Answer C is correct because metabolism by CYP2C9 and CYP3A4 has been shown to be
decreased in patients with CKD (slide 33). CYP2E1 (chlorzoxazone was used as the
example in class; see lecture slide 37) and CYP2C19 (slide 33) are not affected by CKD.
Based on only the drug characteristic specified, which of the following statements
would you predict to be TRUE regarding the expected amount of drug removed
by conventional hemodialysis (i.e., hemodialysis using a cuprophane or cellulose
membrane)? (Note: MW = molecular weight; VD = volume of distribution). The
amount of drug removed by hemodialysis would be GREATER for:
a. foscarnet (MW = 94) than for erythromycin (MW 733.9).
b. ceftriaxone (fraction unbound = 0.10) than for cefazolin (fraction unbound
= 0.50)
c. amitriptyline (VD = 10 L/kg) than trimethoprim (VD = 1.6 L/kg).
d. All of the above are true.
e. A and B
f. A and C
g. B and C
a. foscarnet (MW = 94) than for erythromycin (MW 733.9).

All else being equal, drugs with a smaller MW (especially less than
500) will be removed to a greater extent by hemodialysis than drugs with a larger MW
(especially > 500). Answer B is wrong because drugs that have higher protein binding
(i.e., having a lower fraction unbound) will not be cleared by dialysis as well as a drug
with lower protein binding (i.e., having a higher fraction unbound). Answer C is wrong
because drugs with a large VD (i.e., > 2 L/kg) are not cleared by dialysis as well as
drugs with a smaller VD. Answers D through G are not correct based on the previous
answers.
KM is a 64 year old man (5 ft 10 in; 73 kg) admitted to the hospital for an elective cardiac catheterization. His past medical history includes arthritis (degenerative joint disease), diabetes, and heart failure and his list of medications includes ibuprofen (Motrin), a non-steroidal antiinflammatory agent, metformin (Glucophage), furosemide (Lasix), a K-wasting diuretic, and insulin. His admission labs include the following: Na 142 meq/L, K 5.3 meq/L, Cl 110 meq/L, CO2 24 meq/L, BUN 20 mg/dl, Cr 2.0 mg/dl.

Based only on this information, appropriately categorize this patient’s renal dysfunction.
a. Stage 1 normal; CLCr > 90 ml/min
b. Stage 2 mild; CLCr 60-89 ml/min
c. Stage 3 moderate; CLCr 30-59 ml/min
d. Stage 4 severe; CLCr 15-39 ml/min
c. Stage 3 moderate; CLCr 30-59 ml/min

(Chronic Renal Failure).
IBW = 50 + 2.3 (10) = 73 kg
CLcr = [(140 – 64) (73)] / [(2.0)(72)] = 38.5 ml/min
KM is a 64 year old man (5 ft 10 in; 73 kg) admitted to the hospital for an elective cardiac
catheterization. His past medical history includes arthritis (degenerative joint disease), diabetes,
and heart failure and his list of medications includes ibuprofen (Motrin), a non-steroidal antiinflammatory
agent, metformin (Glucophage), furosemide (Lasix), a K-wasting diuretic, and
insulin. His admission labs include the following: Na 142 meq/L, K 5.3 meq/L, Cl 110 meq/L,
CO2 24 meq/L, BUN 20 mg/dl, Cr 2.0 mg/dl.

Since KM will be receiving contrast media prior to the cardiac catheterization, which of
the following would be most appropriate for preventing contrast media – induced acute
renal dysfunction?
a. all medications should be continued without interruption for maximal renal
protection
b. administer normal saline and N-acetylcysteine the day before and the day of the
catheterization
c. low – dose dopamine, Lasix (furosemide), a diuretic, and fluid restriction the day
before the catheterization
d. contrast media does not adversely affect renal function; no prophylactic measures
are necessary
b. administer normal saline and N-acetylcysteine the day before and the day of the
catheterization

Selection “a” is incorrect since
at least three of the medications KM is taking may cause or exacerbate acute renal failure.
Those medications are ibuprofen, metformin, and Lasix. Selection “c” and “d” are both
incorrect. See pages 789-790 of text for details.
KM is a 64 year old man (5 ft 10 in; 73 kg) admitted to the hospital for an elective cardiac
catheterization. His past medical history includes arthritis (degenerative joint disease), diabetes,
and heart failure and his list of medications includes ibuprofen (Motrin), a non-steroidal antiinflammatory
agent, metformin (Glucophage), furosemide (Lasix), a K-wasting diuretic, and
insulin. His admission labs include the following: Na 142 meq/L, K 5.3 meq/L, Cl 110 meq/L,
CO2 24 meq/L, BUN 20 mg/dl, Cr 2.0 mg/dl.

In spite of appropriate prophylaxis, KM develops acute renal failure 24 hours after the
cardiac catheterization. Which of the following is the most likely etiology in this patient?
a. pre-renal azotemia
b. functional acute renal failure
c. intrinsic acute renal failure
d. post renal obstruction
c. intrinsic acute renal failure

See slide 11 and text pages 782-785 (Table
42-2, page 783 as well).
(Refer to Case C)

PL’s physician wishes to treat his metabolic acidosis with NaHCO3. Which of the
following most correctly characterizes this therapeutic strategy.
a. good idea; NaHCO3 effectively and efficiently lowers serum pH
b. good idea; patient is hypo-natremic
c. bad idea; patient is hyper-natremic
d. bad idea; patient has elevated blood pressure
d. bad idea; patient has elevated blood pressure

Patient’s Na concentration is WNL so
selections b and c are both incorrect. Selection a is incorrect since NaHCO3 raises serum
pH.
(refer to Case C)

PL’s physician wants to administer folic acid for management of his macrocytic anemia.
Which of the following describes most appropriate use of folic acid in this patient?
a. good idea; patient has macrocytic anemia
b. bad idea; patient does not have macrocytic anemia
c. good idea but must administer with vitamin B12
d. bad idea; vitamin B12 is the drug of choice for macrocytic anemia
b. bad idea; patient does not have macrocytic anemia

Since this patient’s MCV is < 100
mm3, he does not have macrocytic anemia.
(Refer to Case C)

Which of the following most closely characterizes the therapeutic goal for managing this
patient’s anemia?
a. Hgb 16 Gm/dl, Hct 48%
b. Hgb 10 Gm/dl, Hct 30%
c. Transferrin saturation < 15%
d. MCV > 100mm3
b. Hgb 10 Gm/dl, Hct 30%

Please see slide 27 and page 826 of text for
additional details.
(refer to Case C)

Which of the following would be most appropriate initial therapy for this patient’s
hyperphosphatemia?
a. aluminum hydroxide gel
b. calcitriol
c. calcium carbonate
d. calcium citrate
a. aluminum hydroxide gel

Since this patient’s corrected Ca
concentration indicates that he is actually hypercalcemic, either Ca salt would be
inappropriate. For the same reason calcitriol would likewise not be useful. Since the
patient’s serum phosphate is > 7.0 mg/dl, the drastic measure of using Al becomes the
best of these available choices.
(refer to Case C)

Which of the following most correctly describes role of cinacelet in the management of
this patient’s hyperphosphatemia?
a. it binds phosphate in the GI tract
b. it enhances renal phosphate excretion
c. it enhances sensitivity of the parathyroid gland to phosphate
d. it enhances sensitivity of the parathyroid gland to Ca
d. it enhances sensitivity of the parathyroid gland to Ca

Please refer to page 840 of text for details of
this agent.
(refer to Case C)

Which of the following represents the best initial therapy of this patient’s hypertension?
a. a calcium channel blocker such as verapamil or diltiazem
b. a thiazide diuretic such as HCTZ
c. an ACE inhibitor such as ramipril or lisinopril
d. hypertension is a benign condition and need not be treated
a. a calcium channel blocker such as verapamil or diltiazem

Thiazide diuretics are not effective in
patients with a creatinine clearance < 30 ml/min and this patient’s approximately 9
ml/min. An ACE inhibitor would potentially exacerbate this patient’s severe
hyperkalemia. I hope that an explanation of selection d is unnecessary. Please see slides
32 and 36
(refer to Case C)

Which of the following most correctly characterizes this patient’s blood pressure goal?
a. < 140/90 mm Hg
b. < 130/80 mm Hg
c. < 125/75 mm Hg
d. > 140/90 mm Hg
b. < 130/80 mm Hg

Even though this
patient’s albumin is quite low, there is no mention of proteinuria. Consequently, his
blood pressure goal is < 130/80.
(refer to Case C)

PL’s physician is forced to treat the hyperkalemia with kayexalate since all hemodialysis
machines have been recalled. Which of the following describes most appropriate use of
this agent in this patient to reduce K to 4.9 meq/L?
a. 15 Gm po x 1 only
b. 50 Gm po x 1 only
c. 50 Gm po qid
d. 200 Gm po x 1
c. 50 Gm po qid

(please see objectives 5 and 6 of CRF and objective 2 of electrolytes).
See slide 26 of Electrolytes and pages 970 and 974-975 of text. Patient has an excess of
approximately 200 meq of K+. Since 1.0 Gm of kayexalate exchanges 1.0 meq of Na+ for
1 meq of K+, a total dose of 200 Gm of kayexalate is needed preferably in divided doses.
Twenty four hours following a renal ultrasound (with intravenous contrast media),
LML’s (60 years old; 6’0”; 190 lbs) urine output has declined to 300 ml/24 hrs.
Also the following labs are noted: Na 142 meq/L, K 4.0 meq/L, CO2 27 meq/L, Cl
105 meq/L, BUN 25 ng/dl, Cr 1.0 mg/dl.

Based on this information, appropriately
stage this patient’s renal dysfunction.
a. 1
b. 2
c. 3
d. 4
b. 2

Patient’s calculated creatinine clearance is
86 ml/min (IBW = 50+2.3 [12] = 77.6 kg; CLcr = [140-60] [78]/[1.0x72] = 86
ml/min) which is consistent with stage 2 or selection “b”.
Twenty four hours following a renal ultrasound (with intravenous contrast media),
LML’s (60 years old; 6’0”; 190 lbs) urine output has declined to 300 ml/24 hrs.
Also the following labs are noted: Na 142 meq/L, K 4.0 meq/L, CO2 27 meq/L, Cl
105 meq/L, BUN 25 ng/dl, Cr 1.0 mg/dl.

LML is felt to have oliguric pre – renal azotemia. Which
of the following would have most likely prevented its occurrence?
a. N-acetylcysteine administered the day before and the day of the
ultrasound
b. Furosemide (Lasix®) and fluid restriction the day before the ultrasound
c. ibuprofen (Motrin®) administered the day before and the day of the
ultrasound
d. contrast media does not adversely affect renal function; no preventative
measures are necessary
a. N-acetylcysteine administered the day before and the day of the
ultrasound

Selections “b” and “c” would
actually increase the likelihood of this problem. Selection “d” is incorrect.
FJ is a 64 year old man with a long (many years) history of chronic kidney disease,
hypertension, and diabetes. During a routine follow-up clinic visit, the following labs
were noted: Na 142 meq/L, K 5.4 meq/L, Cl 105 meq/L, CO2 18 meq/L, BUN 52 mg/dl,
Cr 5.0 mg/dl, Hgb 9.3 Gm/dl, Hct 28%, MCV 125 μm3, Alb 2.7 Gm/dl, Ca 10.5 mg/dl,
phosphate 5.5 mg/dl, Fe 100 mcg/dl, TIBC 300 μg/dl, Transferrin sat’n 40%,
Ferritin 200 mg/ml.
The patient has no specific complaints but his vital signs are as follows: BP 170/100 mm
Hg, P 82, RR16, T 37°C.

Which of the following would be preferred for initial management of this patient’s
macrocytic anemia?
a. begin therapy with Folic Acid only
b. begin therapy with Vitamin B12 only
c. begin therapy with erythropoietin and/or Fe
d. begin therapy with both Folic Acid and Vitamin B12
d. begin therapy with both Folic Acid and Vitamin B12

Selection “a” and “b” are
incorrect since supplementation of folate in a patient with a B12 deficiency will
alleviate the macrocytic anemia but will not prevent the neurologic complications of a B12 deficiency. Selection “c” will not appropriately manage
a macrocytic anemia
FJ is a 64 year old man with a long (many years) history of chronic kidney disease,
hypertension, and diabetes. During a routine follow-up clinic visit, the following labs
were noted: Na 142 meq/L, K 5.4 meq/L, Cl 105 meq/L, CO2 18 meq/L, BUN 52 mg/dl,
Cr 5.0 mg/dl, Hgb 9.3 Gm/dl, Hct 28%, MCV 125 μm3, Alb 2.7 Gm/dl, Ca 10.5 mg/dl,
phosphate 5.5 mg/dl, Fe 100 mcg/dl, TIBC 300 μg/dl, Transferrin sat’n 40%,
Ferritin 200 mg/ml.
The patient has no specific complaints but his vital signs are as follows: BP 170/100 mm
Hg, P 82, RR16, T 37°C.

Which of the following describes most appropriate use of Fe to manage this
patient’s anemia?
a. since this patient’s serum Fe concentration is within normal limits, Fe
therapy should be withheld at this time.
b. begin therapy with oral FeS04 given 2 to 3 times daily.
c. do not administer Fe until resistance to other therapies has been
eliminated.
d. since Transferrin sat’n and serum Ferritin are both within normal limits,
Fe therapy should be withheld at this time.
d. since Transferrin sat’n and serum Ferritin are both within normal limits,
Fe therapy should be withheld at this time.

Selection “a” is
incorrect since serum Fe concentration is an unreliable measure of total
body iron stores. Selection “c” is incorrect since “resistance” is not an
issue for this patient.
FJ is a 64 year old man with a long (many years) history of chronic kidney disease,
hypertension, and diabetes. During a routine follow-up clinic visit, the following labs
were noted: Na 142 meq/L, K 5.4 meq/L, Cl 105 meq/L, CO2 18 meq/L, BUN 52 mg/dl,
Cr 5.0 mg/dl, Hgb 9.3 Gm/dl, Hct 28%, MCV 125 μm3, Alb 2.7 Gm/dl, Ca 10.5 mg/dl,
phosphate 5.5 mg/dl, Fe 100 mcg/dl, TIBC 300 μg/dl, Transferrin sat’n 40%,
Ferritin 200 mg/ml.
The patient has no specific complaints but his vital signs are as follows: BP 170/100 mm
Hg, P 82, RR16, T 37°C.

What is the goal of therapy for management of this patient’s anemia?
a. minimal to no symptoms of anemia; MCV < 100 mm3.
b. minimal to no symptoms of anemia; Hgb 15 Gm/dl; Hct 42%
c. minimal to no symptoms of anemia; Hgb 11 Gm/dl; Hct 33%
d. minimal to no symptoms of anemia; normal Ferritin and Transferrin sat’n
c. minimal to no symptoms of anemia; Hgb 11 Gm/dl; Hct 33%
FJ is a 64 year old man with a long (many years) history of chronic kidney disease,
hypertension, and diabetes. During a routine follow-up clinic visit, the following labs
were noted: Na 142 meq/L, K 5.4 meq/L, Cl 105 meq/L, CO2 18 meq/L, BUN 52 mg/dl,
Cr 5.0 mg/dl, Hgb 9.3 Gm/dl, Hct 28%, MCV 125 μm3, Alb 2.7 Gm/dl, Ca 10.5 mg/dl,
phosphate 5.5 mg/dl, Fe 100 mcg/dl, TIBC 300 μg/dl, Transferrin sat’n 40%,
Ferritin 200 mg/ml.
The patient has no specific complaints but his vital signs are as follows: BP 170/100 mm
Hg, P 82, RR16, T 37°C.

FJ’s physician asks you for a dose recommendation for Aludrox® (aluminium
hydroxide gel) for management of the patient’s renal osteodystrophy. Which of
the following is the most appropriate reply?
a. 15-30 ml po qid
b. Aludrox® is an inappropriate drug for this patient
c. since the patient’s serum Ca is within normal limits, he probably doesn’t
need treatment for renal osteodystrophy
d. since this patient’s serum K is only slightly above normal, he probably
doesn’t need treatment for renal osteodystrophy
b. Aludrox® is an inappropriate drug for this patient

While selection “a” is a reasonable dose for Aludrox®, its use in this patient is
associated with an increased risk of Al-associated encephalopathy.
Selection “c” is incorrect since it fails to account for this patient’s
hypoalbuminemia and hyperphosphatemia. Selection “d” is incorrect
since K is not part of the pathogenesis of renal osteodystrophy.
FJ is a 64 year old man with a long (many years) history of chronic kidney disease,
hypertension, and diabetes. During a routine follow-up clinic visit, the following labs
were noted: Na 142 meq/L, K 5.4 meq/L, Cl 105 meq/L, CO2 18 meq/L, BUN 52 mg/dl,
Cr 5.0 mg/dl, Hgb 9.3 Gm/dl, Hct 28%, MCV 125 μm3, Alb 2.7 Gm/dl, Ca 10.5 mg/dl,
phosphate 5.5 mg/dl, Fe 100 mcg/dl, TIBC 300 μg/dl, Transferrin sat’n 40%,
Ferritin 200 mg/ml.
The patient has no specific complaints but his vital signs are as follows: BP 170/100 mm
Hg, P 82, RR16, T 37°C.

Which of the following represents the best initial therapy for management of this
patient’s hypertension?
a. a diuretic such as hydrochlorothiazide (Esidrix®, others) or triamterene
(Dyrenium®)
b. a diuretic such as furosemide (Lasix®) or bumetanide (Bumex®)
c. ARB such as losartan (Cozaar®) or candesartan (Atacand®)
d. hypertension is a benign condition in this patient and does not need to be
treated.
c. ARB such as losartan (Cozaar®) or candesartan (Atacand®)

Selection “a” is incorrect
since HCTZ is not effective in patient with a creatinine clearance < 30
ml/min. Neither is triamterene but it is also a K-sparing diuretic would
exacerbate this patient’s hyperkalemia. Selection “b” is incorrect since
addition of these agents follows failure of ARB therapy. I hope that an
explanation of “d” is not necessary.
Based on only the drug characteristic specified, which of the following statements
would you predict to be TRUE regarding the expected amount of drug removed
by conventional hemodialysis (i.e., hemodialysis using a cuprophane
membrane)? (Note: MW = molecular weight; VD – volume of distribution).
The amount of drug removed by hemodialysis would be greater for:
a. erythromycin (MW 733.9) than for foscarnet (MW = 94).
b. cefazolin (fraction unbound = 0.50) than for ceftriaxone (fraction unbound =
0.10).
c. amitriptyline (VD = 10 L/kg) than trimethoprim (VD = 1.6 L/kg).
d. All of the above are true.
e. None of the above are true.
b. cefazolin (fraction unbound = 0.50) than for ceftriaxone (fraction unbound =
0.10).

all else being equal, drugs with higher protein binding
(i.e., having a lower fraction unbound) will not be cleared by dialysis as well
as a drug with lower protein binding (i.e., having a higher fraction unbound).
Answer a is not correct as dialysis clearance using a cuprophane membrane
decreases significantly above a MW of 500 and is even more substantially
reduced for MW > 1000. Answers c is not correct because drugs with a large
VD (i.e., > 2 L/kg) are not cleared by dialysis as well as drugs with a smaller
VD. Answers d and e and not correct based on the previous answers.
Tozer Approach to Dosage Individualization
Q = 1 - [fe × (1-KF)]
Q = dosage adjustment factor
fe = fraction eliminated unchanged in urine
KF =CLcrf/CLcrN (CLcrN = 120 mL/min)
Subscripts "N" and "f" refer to patients with normal renal function and renal
failure, respectively.
Equations for Adjustments:
Tf = TN / Q; Df = DN x Q

A 52 year old, 58 kg woman who has a creatinine clearance of 42 ml/min is to
receive valacyclovir for prophylaxis against cytomegalovirus (CMV) infection after
renal transplantation. Valacyclovir is predominantly (75 percent) eliminated
renally unchanged. Based on this information, select the most appropriate
dosing interval for this patient with impaired renal function. Assume the normal
dosing interval (TN) is every 6 hours and that the maximum dosing interval is 24
hours. Select the dosing interval closest to what you calculate.
a. 6 hours
b. 8 hours
c. 12 hours
d. 18 hours
e. 24 hours
c. 12 hours

Calculated as Q = 1-[0.75 × (1 – (42 / 120))] = 0.513
and Tf = TN / Q = 6 / 0.513 = 11.7 hours (closest interval is therefore 12
hours).
Which one of the following statements is TRUE?
a. In general, the volume of distribution of basic drugs is increased in patients
with chronic kidney disease.
b. Total drug concentrations can be used reliably to monitor therapy and make
dose modifications in patients with chronic kidney disease.
c. The clearance of vancomycin by hemodialysis when using synthetic high-flux
hemodialysis filters (e.g., polyacrylonitrile, polymethylmethacrylate, etc.) is
less than the clearance observed with conventional (e.g., cuprophane) filters.
d. Peritoneal dialysis is generally more effective in removing drug substances
than hemodialysis due to the prolonged dialysate dwell times.
e. None of the above; all of the statements are false
e. None of the above; all of the statements are false.

Answer a is not correct because in general, basic drugs
have a smaller volume of distribution (see lecture slide 14). Answer b is not
correct because the relationship between total and free drug concentrations
can be altered in patient with renal kidney disease, which can make total
concentrations unreliable in this patient population (see lecture slides 17 and
23). Answer c is not correct because vancomycin clearance is much higher
with synthetic high-flux filters than with cuprophane filters. Finally, answer d
is not correct because peritoneal dialysis is generally less effective than
hemodialysis in removing substances.
It is now recognized that chronic kidney disease (CKD) can affect hepatic drug
metabolism by decreasing the activity of drug-metabolizing enzymes, such as
cytochrome P450 (CYP) enzymes. Which of the following drugs metabolized by
a CYP enzyme would you LEAST expect to have decreased clearance (due to
decreased enzyme activity) in patients with CKD?
a. Chlorzoxazone (metabolized by CYP2E1)
b. Warfarin (metabolized by CYP2C9)
c. Erythromycin (metabolized by CYP3A4)
d. The clearance of all of these drugs would be decreased in CKD.
e. CKD will not affect the clearance of any of these drugs.
a. Chlorzoxazone (metabolized by CYP2E1)

Answer A is correct because CYP2E1 is not affected by CKD (chlorzoxazone
was used as the example in class; see lecture slide 31). Answers b and c are
not correct since metabolism by CYP2C9 and CYP3A4 has been shown to be decreased in patients with CKD (slide 31). Answers D and E are not correct
based on A-C.
FJ is a 64 year old man with a long (many years) history of chronic kidney disease,
hypertension, and diabetes. During a routine follow-up clinic visit, the following labs
were noted: Na 142 meq/L, K 5.4 meq/L, Cl 105 meq/L, CO2 18 meq/L, BUN 20 mg/dl,
Cr 2.5 mg/dl, Hgb 9.3 Gm/dl, Hct 28%, MCV 82 μm3, Alb 2.7Gm/dl, Ca 10.5 mg/dl,
phosphate 5.5 mg/dl, Fe 90 mcg/dl, TIBC 460 μg/dl, Transferrin sat’n 14%, Ferritin 15
ng/ml.
The patient has no specific complaints but his vital signs are as follows: BP 170/100 mm
Hg, P 82, RR16, T 37°C.

Which of the following represents most appropriate therapy for this patient’s
metabolic acidosis?
a. patient should preferably be treated with either Shohl’s solution (citric
acid/Na citrate) or Bicitra (citric acid/Na citrate)
b. patient should preferably be treated with Polycitra (citric acid/Na citrate/K
citrate).
c. patient should preferably be treated with NaHCO3.
d. patient does not yet have metabolic acidosis; no therapy is necessary yet.
a. patient should preferably be treated with either Shohl’s solution (citric
acid/Na citrate) or Bicitra (citric acid/Na citrate)

Selection “b”
is incorrect due to the K content of this product and the patient already has mild
hyperkalemia. Selection “c” is incorrect since the patient has HTN and the Na
content of NaHCO3 would likely exacerbate this patient’s already elevated blood
pressure. Selection “d” is incorrect by definition of metabolic acidosis (see slide
10).
FJ is a 64 year old man with a long (many years) history of chronic kidney disease,
hypertension, and diabetes. During a routine follow-up clinic visit, the following labs
were noted: Na 142 meq/L, K 5.4 meq/L, Cl 105 meq/L, CO2 18 meq/L, BUN 20 mg/dl,
Cr 2.5 mg/dl, Hgb 9.3 Gm/dl, Hct 28%, MCV 82 μm3, Alb 2.7Gm/dl, Ca 10.5 mg/dl,
phosphate 5.5 mg/dl, Fe 90 mcg/dl, TIBC 460 μg/dl, Transferrin sat’n 14%, Ferritin 15
ng/ml.
The patient has no specific complaints but his vital signs are as follows: BP 170/100 mm
Hg, P 82, RR16, T 37°C.

Which of the following would be preferred for initial management of this
patient’s anemia?
a. since this patient probably has a macrocytic anemia, begin therapy with
Folic Acid.
b. since this patient probably has a macrocytic anemia, begin therapy with
vitamin B12.
c. begin therapy with erythropoietin or darbepoietin.
d. since this patient has no symptoms typically associated with anemia, no
therapy for anemia is appropriate at this time.
c. begin therapy with erythropoietin or darbepoietin.

Selections “a”
and “b” are both incorrect since this patient does not have a macrocytic anemia
by virtue of an MCV that is within normal limits. Selection “d” is incorrect since
absence of symptoms is not the primary determinant of initiating therapy (see
slide #19).
FJ is a 64 year old man with a long (many years) history of chronic kidney disease,
hypertension, and diabetes. During a routine follow-up clinic visit, the following labs
were noted: Na 142 meq/L, K 5.4 meq/L, Cl 105 meq/L, CO2 18 meq/L, BUN 20 mg/dl,
Cr 2.5 mg/dl, Hgb 9.3 Gm/dl, Hct 28%, MCV 82 μm3, Alb 2.7Gm/dl, Ca 10.5 mg/dl,
phosphate 5.5 mg/dl, Fe 90 mcg/dl, TIBC 460 μg/dl, Transferrin sat’n 14%, Ferritin 15
ng/ml.
The patient has no specific complaints but his vital signs are as follows: BP 170/100 mm
Hg, P 82, RR16, T 37°C.

What is the goal of therapy for management of this patient’s anemia?
a. minimal to no symptoms of anemia; MCV > 100 mm3.
b. minimal to no symptoms of anemia; Hgb 15 Gm/dl; Hct 42%
c. minimal to no symptoms of anemia; Hgb 11 Gm/dl; Hct 33%
d. minimal to no symptoms of anemia; normal Ferritin and Transferrin sat’n
c. minimal to no symptoms of anemia; Hgb 11 Gm/dl; Hct 33%

please see objective 7 and see slide 17.
FJ is a 64 year old man with a long (many years) history of chronic kidney disease,
hypertension, and diabetes. During a routine follow-up clinic visit, the following labs
were noted: Na 142 meq/L, K 5.4 meq/L, Cl 105 meq/L, CO2 18 meq/L, BUN 20 mg/dl,
Cr 2.5 mg/dl, Hgb 9.3 Gm/dl, Hct 28%, MCV 82 μm3, Alb 2.7Gm/dl, Ca 10.5 mg/dl,
phosphate 5.5 mg/dl, Fe 90 mcg/dl, TIBC 460 μg/dl, Transferrin sat’n 14%, Ferritin 15
ng/ml.
The patient has no specific complaints but his vital signs are as follows: BP 170/100 mm
Hg, P 82, RR16, T 37°C.

Which of the following describes most appropriate use of Fe to manage this
patient’s anemia?
a. since this patient’s serum Fe concentration is within normal limits, Fe
therapy should be withheld at this time.
b. begin therapy with oral FeS04 given 2 to 3 times daily.
c. do not administer Fe until resistance to other therapies has been
eliminated.
d. since Transferrin sat’n and serum Ferritin are both within normal limits,
Fe therapy should be withheld at this time.
b. begin therapy with oral FeS04 given 2 to 3 times daily

Selection “a”
is incorrect since serum Fe concentration is an unreliable measure of total body
iron stores. Selection “d” is incorrect even though these values are within normal
limits. Please note that both Transferrin sat’n and Ferritin are low normal. It is
quite likely that initiation of EPO or darbepoeitin will use up Fe stores and
response to these agents would be limited as a result. Selection “c” is incorrect
since Fe stores should ideally be optimized before initiation of EPO or
darbepoeitin ( see slide #19).
FJ is a 64 year old man with a long (many years) history of chronic kidney disease,
hypertension, and diabetes. During a routine follow-up clinic visit, the following labs
were noted: Na 142 meq/L, K 5.4 meq/L, Cl 105 meq/L, CO2 18 meq/L, BUN 20 mg/dl,
Cr 2.5 mg/dl, Hgb 9.3 Gm/dl, Hct 28%, MCV 82 μm3, Alb 2.7Gm/dl, Ca 10.5 mg/dl,
phosphate 5.5 mg/dl, Fe 90 mcg/dl, TIBC 460 μg/dl, Transferrin sat’n 14%, Ferritin 15
ng/ml.
The patient has no specific complaints but his vital signs are as follows: BP 170/100 mm
Hg, P 82, RR16, T 37°C.

Which of the following would be most appropriate initial therapy for treatment of
this patient’s renal osteodystrophy?
a. Sevelamer (Renagel)
b. Ca CO3 (Tums, others)
c. Al/Mg hydroxide suspension (Maalox, Mylanta, others), an antacid
d. since this patient’s serum Ca is within normal limits, he probably does not
have renal osteodystrophy yet.
a. Sevelamer (Renagel)

Although the
patient’s observed Ca is within normal limits, the corrected Ca (i.e. 11.5 mg/dl) is
not. Also, the Ca – phosphate product is > 55 (i.e. 64) which strongly suggests
that this patient does have renal osteodystrophy which renders selection “d”
incorrect. Selection “b” is incorrect by virtue of this patient’s corrected
hypercalcemia. Selection “c” is incorrect since use of Mg salts is patients with
chronic kidney disease may lead to hypermagnesemia and/or aluminum toxicity.
FJ is a 64 year old man with a long (many years) history of chronic kidney disease,
hypertension, and diabetes. During a routine follow-up clinic visit, the following labs
were noted: Na 142 meq/L, K 5.4 meq/L, Cl 105 meq/L, CO2 18 meq/L, BUN 20 mg/dl,
Cr 2.5 mg/dl, Hgb 9.3 Gm/dl, Hct 28%, MCV 82 μm3, Alb 2.7Gm/dl, Ca 10.5 mg/dl,
phosphate 5.5 mg/dl, Fe 90 mcg/dl, TIBC 460 μg/dl, Transferrin sat’n 14%, Ferritin 15
ng/ml.
The patient has no specific complaints but his vital signs are as follows: BP 170/100 mm
Hg, P 82, RR16, T 37°C.

Which of the following correctly characterizes use of vitamin D for management
of this patient’s renal osteodystrophy.
a. its use should be withheld until phosphate concentration has been
normalized.
b. begin vitamin D capsules daily immediately.
c. begin calcitriol (Rocaltrol) immediately.
d. Vitamin D must be used together with Ca CO3 in this patient.
a. its use should be withheld until phosphate concentration has been
normalized.

Both “b” and “c” are incorrect
since phosphate must be reduced first. Vitamin D, or more correctly, calcitriol will
also raise phosphate levels and the present hyperphosphatemia must be normalized before beginning such therapy. Selection “b” is also incorrect since
the patient will likely be unable to produce the 1, 25 dihydroxy derivative of
vitamin D as a consequence of his chronic kidney disease. Selection “d” is
incorrect also because vitamin D will likely be inactive but also because this
patient is already hypercalcemic and will likely not benefit from exogenous Ca at
the present time.
FJ is a 64 year old man with a long (many years) history of chronic kidney disease,
hypertension, and diabetes. During a routine follow-up clinic visit, the following labs
were noted: Na 142 meq/L, K 5.4 meq/L, Cl 105 meq/L, CO2 18 meq/L, BUN 20 mg/dl,
Cr 2.5 mg/dl, Hgb 9.3 Gm/dl, Hct 28%, MCV 82 μm3, Alb 2.7Gm/dl, Ca 10.5 mg/dl,
phosphate 5.5 mg/dl, Fe 90 mcg/dl, TIBC 460 μg/dl, Transferrin sat’n 14%, Ferritin 15
ng/ml.
The patient has no specific complaints but his vital signs are as follows: BP 170/100 mm
Hg, P 82, RR16, T 37°C.

Which of the following represent the best initial therapy for management of this
patient’s hypertension?
a. a calcium channel blocker such as verapamil (Calan) or diltiazem
(Cardizem).
b. a beta blocker such as nadolol (Corgard) or atenolol (Tenormin).
c. an ACE inhibitor such as ramipril (Altace) or lisinopril (Zestril)
d. hypertension is a benign condition in this patient and does not need to be
treated.
c. an ACE inhibitor such as ramipril (Altace) or lisinopril (Zestril)

Selection “a” represents a
reasonable antihypertensive strategy but not for initial therapy. Please see slides
23 and 24. I would hope that an explanation of selection “d” would not be
necessary.
FJ is a 64 year old man with a long (many years) history of chronic kidney disease,
hypertension, and diabetes. During a routine follow-up clinic visit, the following labs
were noted: Na 142 meq/L, K 5.4 meq/L, Cl 105 meq/L, CO2 18 meq/L, BUN 20 mg/dl,
Cr 2.5 mg/dl, Hgb 9.3 Gm/dl, Hct 28%, MCV 82 μm3, Alb 2.7Gm/dl, Ca 10.5 mg/dl,
phosphate 5.5 mg/dl, Fe 90 mcg/dl, TIBC 460 μg/dl, Transferrin sat’n 14%, Ferritin 15
ng/ml.
The patient has no specific complaints but his vital signs are as follows: BP 170/100 mm
Hg, P 82, RR16, T 37°C.

Which of the following describes appropriate use of anti-hypertensive
medication(s) for this patient?
a. an angiotensin receptor blocker, such as candesartan (Atacand), is
clearly preferred over an ACE inhibitor such as ramipril (Altace).
b. a K – sparing diuretic, such as spironolactone (Aldactone), is clearly
preferred over a K- wasting diuretic such as furosemide (Lasix).
c. use of a beta blocker, such as metoprolol (Lopressor), may be a
reasonable choice even though it may exacerbate renal dysfunction.
d. use of a beta blocker, such as metoprolol (Lopressor), is clearly
contraindicated since it may exacerbate renal dysfunction.
c. use of a beta blocker, such as metoprolol (Lopressor), may be a
reasonable choice even though it may exacerbate renal dysfunction.

Use of an ARB is
appropriate in the patient who is intolerant to an ACE inhibitor but it is not yet
preferred over an ACE inhibitor. Consequently, selection “a” is incorrect.
Selection “b” is incorrect since a K – sparing diuretic in this patient may
exacerbate his hyperkalemia. Selection “d” is incorrect since the benefits of
beta-blockade for exceed this largely theoretical risk.
Hemodialysis

B.L. is a 72 year old, 65kg man. He presents to his primary care physician with complaints of weakness, lack of energy, and productive cough with yellow
sputum. The treatment selected is oral ciprofloxacin. The usual dose of
ciprofloxacin is 500 mg BID for patients with normal renal function, approximately
60% of ciprofloxacin is eliminated renally unchanged and this patient’s creatinine
clearance is 35 ml/min. Based on this information, calculate the dosage
adjustment factor (Q) that could be used to adjust the treatment regimen for B.L.
a. 0.283
b. 0.425
c. 0.575
d. 0.710
e. 0.825
c. 0.575
Hemodialysis

Which of the following drugs is LEAST likely to be removed by conventional
hemodialysis (i.e., hemodialysis using a celluose membrane)?
a. Foscarnet (MW = 94; Vd = 0.7 l/kg; Fraction bound = 0.17)
b. Cefazolin (MW = 454; Vd = 0.2 l/kg; Fraction bound = 0.50)
c. Ceftriaxone (MW = 450; Vd = 0.2 l/kg; Fraction bound = 0.90)
d. Inulin (MW = 5200; Vd = 0.05 l/kg; Fraction bound = 0.00)
d. Inulin (MW = 5200; Vd = 0.05 l/kg; Fraction bound = 0.00)
It is now recognized that chronic kidney disease (CKD) can affect hepatic drug
metabolism by decreasing the activity of drug-metabolizing enzymes, such as
cytochrome P450 (CYP) enzymes. Which of the following drugs metabolized by
a CYP enzyme would you MOST expect to have decreased clearance (due to
decreased enzyme activity) in patients with CKD?
a. Caffeine (metabolized by CYP1A2)
b. Warfarin (metabolized by CYP2C9)
c. Midazolam (metabolized by CYP3A4)
d. The clearance of all of these drugs would be decreased in CKD.
e. CKD will not affect the clearance of any of these drugs.
b. Warfarin (metabolized by CYP2C9)